Diễn Đàn MathScopeDiễn Đàn MathScope
  Diễn Đàn MathScope
Ghi Danh Hỏi/Ðáp Thành Viên Social Groups Lịch Ðánh Dấu Ðã Ðọc

Go Back   Diễn Đàn MathScope > Sơ Cấp > Việt Nam và IMO > 2012

News & Announcements

Ngoài một số quy định đã được nêu trong phần Quy định của Ghi Danh , mọi người tranh thủ bỏ ra 5 phút để đọc thêm một số Quy định sau để khỏi bị treo nick ở MathScope nhé !

* Nội quy MathScope.Org

* Một số quy định chung !

* Quy định về việc viết bài trong diễn đàn MathScope

* Nếu bạn muốn gia nhập đội ngũ BQT thì vui lòng tham gia tại đây

* Những câu hỏi thường gặp

* Về việc viết bài trong Box Đại học và Sau đại học


Trả lời Gởi Ðề Tài Mới
 
Ðiều Chỉnh Xếp Bài
Old 17-04-2012, 12:16 PM   #16
hqdhftw
+Thành Viên+
 
Tham gia ngày: Sep 2011
Bài gởi: 133
Thanks: 81
Thanked 153 Times in 80 Posts
Theo mình câu b bài 2 đáp số không phải là n đâu, chẳng hạn có thể lắp 6 máy bơm tưới cả cánh đồng 3x7(file đính kèm)
[RIGHT][I][B]Nguồn: MathScope.ORG[/B][/I][/RIGHT]
 
Hình Kèm Theo
Kiểu File : jpg Untitled.jpg (19.4 KB, 95 lần tải)
hqdhftw is offline   Trả Lời Với Trích Dẫn
The Following 2 Users Say Thank You to hqdhftw For This Useful Post:
conami (17-04-2012), kien10a1 (17-04-2012)
Old 17-04-2012, 01:49 PM   #17
pte.alpha
+Thành Viên+
 
Tham gia ngày: Apr 2009
Bài gởi: 216
Thanks: 8
Thanked 208 Times in 62 Posts
Trích:
Nguyên văn bởi mathstarofvn View Post
Theo mình bài toán tổ hợp phải hiểu là chúng ta chẳng cần tưới nước ô đặt máy bơm, đôi khi phải thực tế 1 tí là chẳng ai cần thiết đi bơm nước vào chỗ đặt máy bơm cả, như thế bài toán mới có vấn đề để nói và câu b đáp số ko còn là n nữa
Đề thi nói rõ là máy bơm có thể tưới nước cho ô chứa nó, các ô cạnh nó và các ô trên cùng cột cách 1 ô.
[RIGHT][I][B]Nguồn: MathScope.ORG[/B][/I][/RIGHT]
 
pte.alpha is offline   Trả Lời Với Trích Dẫn
Old 17-04-2012, 03:59 PM   #18
huynhcongbang
Administrator

 
huynhcongbang's Avatar
 
Tham gia ngày: Feb 2009
Đến từ: Ho Chi Minh City
Bài gởi: 2,413
Thanks: 2,165
Thanked 4,188 Times in 1,381 Posts
Gửi tin nhắn qua Yahoo chát tới huynhcongbang
Ngày 2 có 3 câu như sau:
- Câu 4: dãy số nguyên (tương tự bài 6 VMO 2012).
- Câu 5: BĐT.
- Câu 6: tổ hợp liên quan đến graph.

Mình mới hỏi được nội dung cụ thể của câu BĐT như sau:

Cho 17 số thực dương $a_1,a_2,...,a_{17} $ thỏa mãn điều kiện:

$a_1^2+a_2^2+a_3^2+...+a_{17}^2=21 $.

Chứng minh rằng $C=10\sqrt{24} $ là hằng số lớn nhất để nếu có
$\sum_{i=1}^{17}(a_i^3+a_i) \le C $
thì 3 số bất kì trong 17 số đã cho lập thành 3 cạnh của tam giác.
[RIGHT][I][B]Nguồn: MathScope.ORG[/B][/I][/RIGHT]
 
__________________
Sự im lặng của bầy mèo
huynhcongbang is offline   Trả Lời Với Trích Dẫn
The Following User Says Thank You to huynhcongbang For This Useful Post:
n.v.thanh (17-04-2012)
Old 17-04-2012, 04:15 PM   #19
hoangcongduc
+Thành Viên+
 
Tham gia ngày: Dec 2009
Đến từ: THPT chuyên Lý Tự Trọng Cần Thơ
Bài gởi: 78
Thanks: 92
Thanked 64 Times in 41 Posts
Gửi tin nhắn qua Yahoo chát tới hoangcongduc
Anh Lữ nhầm rồi ạ. Đề ngày 2 thế này:

Bài 4: (7 điểm)
Cho dãy số $(x_n)$ xác định bởi $x_1=1,x_2=2011$ và $x_{n+2}=4022x_{n+1}-x_n,\forall n\in \mathbb N$.
Chứng minh rằng $\dfrac{x_{2012}+1}{2012}$ là số chính phương.

Bài 5: (7 điểm)
Chứng minh rằng $c=10\sqrt{24}$ là hằng số lớn nhất thỏa mãn điều kiện: nếu có các số dương $a_1,a_2,...a_{17}$ sao cho: $$\sum_{i=1}^{17}{a_i^2}=24\qquad ;\qquad \sum_{i=1}^{17}{a_i^3}+\sum_{i=1}^{17}{a_i}<c$$ Thì với mọi $i,j,k$ thỏa mãn $1\le i<j<k\le 17$, ta luôn có $a_i,a_j,a_k$ là độ dài ba cạnh của một tam giác.

Bài 6: (7 điểm)
Có 42 học sinh tham dự kì thi chọn đội tuyển Olympic toán quốc tế. Biết rằng một học sinh bất kì quen đúng 20 học sinh khác. Chứng minh rằng ta có thể chia 42 học sinh thành 2 nhóm hoặc 21 nhóm sao cho số học sinh trong các nhóm bằng nhau và 2 học sinh bất kì trong cùng nhóm thì quen nhau.

Tất nhiên đây không phải là nguyên văn
[RIGHT][I][B]Nguồn: MathScope.ORG[/B][/I][/RIGHT]
 
__________________
-----------------
-------------------------
TIÊN HỌC LỄ HẬU HỌC THÊM

thay đổi nội dung bởi: hoangcongduc, 17-04-2012 lúc 04:55 PM
hoangcongduc is offline   Trả Lời Với Trích Dẫn
The Following 9 Users Say Thank You to hoangcongduc For This Useful Post:
huynhcongbang (17-04-2012), Lan Phuog (18-04-2012), n.v.thanh (17-04-2012), nyctkt (17-04-2012), pte.alpha (17-04-2012), sine (17-04-2012), thefallen (17-04-2012), tramy_hanoi (17-04-2012), trang96 (17-04-2012)
Old 17-04-2012, 04:31 PM   #20
kien10a1
+Thành Viên+
 
kien10a1's Avatar
 
Tham gia ngày: Feb 2011
Đến từ: Vĩnh Yên- Vĩnh Phúc
Bài gởi: 371
Thanks: 43
Thanked 263 Times in 153 Posts
Gửi tin nhắn qua Yahoo chát tới kien10a1
Trích:
Nguyên văn bởi hqdhftw View Post
Theo mình câu b bài 2 đáp số không phải là n đâu, chẳng hạn có thể lắp 6 máy bơm tưới cả cánh đồng 3x7(file đính kèm)
Đúng rồi, mình bị nhầm mất đoạn cuối, lập luận đó chỉ suy ra 5 cột từ k đến k+4 phải có ít nhất 4 máy.
[RIGHT][I][B]Nguồn: MathScope.ORG[/B][/I][/RIGHT]
 
__________________
Quay về với nơi bắt đầu
kien10a1 is offline   Trả Lời Với Trích Dẫn
Old 17-04-2012, 04:52 PM   #21
CSS-MU
+Thành Viên+
 
Tham gia ngày: Mar 2010
Bài gởi: 26
Thanks: 2
Thanked 100 Times in 16 Posts
Gửi mọi người file PDF đề 2 ngày thi.
[RIGHT][I][B]Nguồn: MathScope.ORG[/B][/I][/RIGHT]
 
File Kèm Theo
Kiểu File : pdf VN TST2012 (Day 1 and 2).pdf (128.1 KB, 450 lần tải)
__________________
Đời vô đối...

thay đổi nội dung bởi: CSS-MU, 17-04-2012 lúc 08:05 PM
CSS-MU is offline   Trả Lời Với Trích Dẫn
The Following 16 Users Say Thank You to CSS-MU For This Useful Post:
A Good Man (17-04-2012), coban (17-04-2012), conami (17-04-2012), lovemath_ltv (17-04-2012), Mathpro123 (18-04-2012), muamuathu123 (17-04-2012), n.v.thanh (17-04-2012), ngocson_dhsp (17-04-2012), nguyentrai_oly (09-06-2012), nyctkt (17-04-2012), perfectstrong (17-04-2012), Shuichi Akai (03-05-2012), sine (17-04-2012), thanhgand (19-04-2012), TKT (18-04-2012), vô_ngã (17-08-2012)
Old 17-04-2012, 07:32 PM   #22
huynhcongbang
Administrator

 
huynhcongbang's Avatar
 
Tham gia ngày: Feb 2009
Đến từ: Ho Chi Minh City
Bài gởi: 2,413
Thanks: 2,165
Thanked 4,188 Times in 1,381 Posts
Gửi tin nhắn qua Yahoo chát tới huynhcongbang
Bài 4 mình đang thử chứng minh bài toán tổng quát hơn là:

Cho p là số nguyên tố lẻ. Xét dãy số $(x_n) $ xác định bởi:

$x_1=1,x_2=p,x_{n+2}=2px_{n+1}-x_n, n \ge 1 $.

Chứng minh rằng $\frac{x_{p+1}+1}{p+1} $ là số chính phương.

Thử với $p=3,5 $ thì đúng rồi.
[RIGHT][I][B]Nguồn: MathScope.ORG[/B][/I][/RIGHT]
 
__________________
Sự im lặng của bầy mèo
huynhcongbang is offline   Trả Lời Với Trích Dẫn
The Following 3 Users Say Thank You to huynhcongbang For This Useful Post:
n.v.thanh (17-04-2012), ngocson_dhsp (17-04-2012), perfectstrong (17-04-2012)
Old 17-04-2012, 08:10 PM   #23
leviethai
+Thành Viên+
 
Tham gia ngày: Nov 2008
Đến từ: Thành phố Hồ Chí Minh. Nhưng quê tôi là Ninh Bình.
Bài gởi: 513
Thanks: 121
Thanked 787 Times in 349 Posts
Gửi tin nhắn qua Yahoo chát tới leviethai
Trích:
Nguyên văn bởi huynhcongbang View Post
Bài 4 mình đang thử chứng minh bài toán tổng quát hơn là:

Cho p là số nguyên tố lẻ. Xét dãy số $(x_n) $ xác định bởi:

$x_1=1,x_2=p,x_{n+2}=2px_{n+1}-x_n, n \ge 1 $.

Chứng minh rằng $\frac{x_{p+1}+1}{p+1} $ là số chính phương.

Thử với $p=3,5 $ thì đúng rồi.
Hình như dự đoán này là đúng.

Không khó để chứng minh công thức tường minh của $x_{n+1} $ là
$x_{n+1}=\dfrac{a^n+b^n}{2}. $
với $a=p+\sqrt{p^2-1},\;b=p-\sqrt{p^2-1}, $ $a+b=2p,\;ab=1. $

Khi đó
$\[P = \frac{{{x_{p + 1}} + 1}}{{p + 1}} = \frac{{{{({a^{\frac{p}{2}}} + {b^{\frac{p}{2}}})}^2}}}{{2(p + 1)}}.\] $

Để ý, $\sqrt{a}+\sqrt{b}=\sqrt{2(p+1)},\;\sqrt{ab}=1, $ khi đó không khó để chứng minh (quy nạp là được)
$\[{a^{\frac{p}{2}}} + {b^{\frac{p}{2}}} = x\sqrt {2(p + 1)} .\] $
(để ý $p $ là số lẻ, $x $ là một số nguyên dương, để rõ ràng thì đặt $m=\sqrt{a},\;n=\sqrt{b} $)

Ta suy ra ngay điều phải chứng minh.
[RIGHT][I][B]Nguồn: MathScope.ORG[/B][/I][/RIGHT]
 
leviethai is offline   Trả Lời Với Trích Dẫn
The Following 3 Users Say Thank You to leviethai For This Useful Post:
huynhcongbang (17-04-2012), perfectstrong (17-04-2012), thefallen (17-04-2012)
Old 17-04-2012, 08:20 PM   #24
kien10a1
+Thành Viên+
 
kien10a1's Avatar
 
Tham gia ngày: Feb 2011
Đến từ: Vĩnh Yên- Vĩnh Phúc
Bài gởi: 371
Thanks: 43
Thanked 263 Times in 153 Posts
Gửi tin nhắn qua Yahoo chát tới kien10a1
Em thử xem nào: $x_{n+2}=2p(2px_n - x_{n-1})-x_n=(4p^2-2)x_n-x_{n-2} $
Xét dãy $(y_n): y_1=1;y_2=2p-1; y_{n+1}=2py_n-y_{n-1} $
Ta chứng minh bằng quy nạp rằng $(p+1)(y_n)^2+1=x_{2n} $
Thật vậy, từ cách cho $y_n $ có
$y_n^2-y_{n-1}y_{n+1}=2-2p $
$(y_{n+1}+y_{n-1})^2=4p^2y_n^2\Rightarrow y_{n+1}^2=4p^2y_n^2-y_{n-1}^2-2y_{n-1}y_{n+1}=(4p^2-2)y_n^2-y_{n-1}^2 -2(2p-2) $
$\Rightarrow y_{n+1}=\frac{(4p^2-2)(x_{2n}+1)}{p+1}-\frac{x_{2n-2}+1}{p+1}+2(2-2p)=\frac{x_{2n+2}+1}{p+1} $
Suy ra $\frac{x_{2n}+1}{p+1} $ chính phương với mọi n, p lẻ nên có đpcm.
[RIGHT][I][B]Nguồn: MathScope.ORG[/B][/I][/RIGHT]
 
__________________
Quay về với nơi bắt đầu

thay đổi nội dung bởi: congbang_dhsp, 17-04-2012 lúc 11:07 PM
kien10a1 is offline   Trả Lời Với Trích Dẫn
The Following 4 Users Say Thank You to kien10a1 For This Useful Post:
Aponium (18-04-2012), huynhcongbang (17-04-2012), minh_thương911 (17-04-2012), thefallen (17-04-2012)
Old 17-04-2012, 08:35 PM   #25
huynhcongbang
Administrator

 
huynhcongbang's Avatar
 
Tham gia ngày: Feb 2009
Đến từ: Ho Chi Minh City
Bài gởi: 2,413
Thanks: 2,165
Thanked 4,188 Times in 1,381 Posts
Gửi tin nhắn qua Yahoo chát tới huynhcongbang
Hãy so sánh bài 5 với bài thi IMO 2004 và bài China MO 1988 dưới đây:

http://www.artofproblemsolving.com/F...55837ee#p99756

Đây là bài giải của anh Cẩn, khá nhanh gọn:

http://diendantoanhoc.net/forum/inde...t=0entry311104
[RIGHT][I][B]Nguồn: MathScope.ORG[/B][/I][/RIGHT]
 
__________________
Sự im lặng của bầy mèo
huynhcongbang is offline   Trả Lời Với Trích Dẫn
Old 17-04-2012, 08:57 PM   #26
nghiepdu-socap
+Thành Viên+
 
nghiepdu-socap's Avatar
 
Tham gia ngày: Apr 2010
Bài gởi: 193
Thanks: 195
Thanked 129 Times in 72 Posts
Bài 4 có cách khác ạ:
$x_{n+2}.x_n=x_{n+1}^2+p^2-1 \Rightarrow \frac{(x_{n+2}+1)(x_n+1)}{(p+1)^2}=\frac{(x_{n+1}+ p)^2}{(p+1)^2} $
Dễ dàng chứng minh được giá trị này là số chính phương nên quy nạp suy ra đpcm
[RIGHT][I][B]Nguồn: MathScope.ORG[/B][/I][/RIGHT]
 
nghiepdu-socap is offline   Trả Lời Với Trích Dẫn
The Following User Says Thank You to nghiepdu-socap For This Useful Post:
huynhcongbang (20-04-2012)
Old 17-04-2012, 09:11 PM   #27
chemgiodaihiep
+Thành Viên+
 
Tham gia ngày: Apr 2010
Bài gởi: 16
Thanks: 10
Thanked 13 Times in 9 Posts
Bài 1 ngày 2 bên Diendantoanhoc mình thấy có lời giải của anh Cẩn đó.
[Only registered and activated users can see links. ]
[RIGHT][I][B]Nguồn: MathScope.ORG[/B][/I][/RIGHT]
 
__________________
-Thành-
chemgiodaihiep is offline   Trả Lời Với Trích Dẫn
Old 17-04-2012, 10:07 PM   #28
ThangToan
+Thành Viên+
 
Tham gia ngày: Nov 2010
Đến từ: THPT chuyên Vĩnh Phúc
Bài gởi: 570
Thanks: 24
Thanked 537 Times in 263 Posts
Trích:
Nguyên văn bởi hoangcongduc View Post

Bài 4: (7 điểm)
Cho dãy số $(x_n)$ xác định bởi $x_1=1,x_2=2011$ và $x_{n+2}=4022x_{n+1}-x_n,\forall n\in \mathbb N$.
Chứng minh rằng $\dfrac{x_{2012}+1}{2012}$ là số chính phương.
Ta thay $2011 $ bởi số nguyên tố
Bài 4.1: (7 điểm)
Cho dãy số $(x_n)$ xác định bởi $x_1=1,x_2=p$ và $x_{n+2}=2px_{n+1}-x_n,\forall n\in \mathbb N$.
Chứng minh rằng $\dfrac{x_{p+1}+1}{p+1}$ là số chính phương.
Chứng minh. Trước hết ta xét tỷ số $\frac{x_n+1}{p+1} $, khi thay $n=1,2,...,6 $ ta được:
$\frac{x_2+1}{p+1}=1, \frac{x_4+1}{p+1}=(2p-1)^2, \frac{x_6+1}{p+1}=(4p^2-2p-1)^2 $. Do đó ta dự đoán $\frac{x_{2n}+1}{p+1}=y_n^2 $, trong đó $y_1=1, y_2=2p-1, y_3=4p^2-2p-1 $. Ta sẽ chọn dãy $(y_n) $ dưới dạng tuyến tính $y_{n+2}=ay_{n+1}+by_n $, thay $n=1 $ vào đẳng thức này ta được: $y_3=ay_2+by_1 $ hay $4p^2-2p-1=a(2p-1)+b $ nên ta chọn $a=2p, b=-1 $. Khi đó ta được dãy $(y_n) $ được xác định như sau:
$y_1=1, y_2=2p-1, y_{n+2}=2py_{n+1}-y_n $.
Tiếp theo ta chứng minh bằng quy nạp đẳng thức:
$\frac{x_{2n}+1}{p+1}=y_n^2, \forall n\ge 1 $
[RIGHT][I][B]Nguồn: MathScope.ORG[/B][/I][/RIGHT]
 
ThangToan is offline   Trả Lời Với Trích Dẫn
The Following 3 Users Say Thank You to ThangToan For This Useful Post:
huynhcongbang (20-04-2012), ngocson_dhsp (17-04-2012), perfectstrong (17-04-2012)
Old 17-04-2012, 11:31 PM   #29
ThangToan
+Thành Viên+
 
Tham gia ngày: Nov 2010
Đến từ: THPT chuyên Vĩnh Phúc
Bài gởi: 570
Thanks: 24
Thanked 537 Times in 263 Posts
Trích:
Nguyên văn bởi hoangcongduc View Post

Bài 6: (7 điểm)
Có 42 học sinh tham dự kì thi chọn đội tuyển Olympic toán quốc tế. Biết rằng một học sinh bất kì quen đúng 20 học sinh khác. Chứng minh rằng ta có thể chia 42 học sinh thành 2 nhóm hoặc 21 nhóm sao cho số học sinh trong các nhóm bằng nhau và 2 học sinh bất kì trong cùng nhóm thì quen nhau.
Có lẽ bài này là kết quả mở rộng của định lý liên quan đến cặp ghép sau:
Cho đồ thị vô hướng $G=(V, E) $ với $|X|\ge 2n $ và bậc của mỗi đỉnh của đồ thị đều không nhỏ hơn $n $. Khi đó trong $G $ luôn tồn tại một đồ thị bộ phận hai mảng $G_1=(V_1, V_2, E_1) $ ($E_1 $ là tập con của $E $) với $|V_1|=n $ đồng thời ghép cặp được $V_1 $ với $V_2 $ (trên đồ thị bộ phận)
[RIGHT][I][B]Nguồn: MathScope.ORG[/B][/I][/RIGHT]
 
ThangToan is offline   Trả Lời Với Trích Dẫn
Old 18-04-2012, 02:41 AM   #30
NhamNgaHanh
Vọng Phong Nhi Đào
 
NhamNgaHanh's Avatar
 
Tham gia ngày: Jul 2011
Bài gởi: 282
Thanks: 85
Thanked 207 Times in 111 Posts
Năm nay có hai bài tìm, nhưng lại cho biết kết quả.
[RIGHT][I][B]Nguồn: MathScope.ORG[/B][/I][/RIGHT]
 
__________________
Nhâm Ngã Hành
NhamNgaHanh is offline   Trả Lời Với Trích Dẫn
Trả lời Gởi Ðề Tài Mới

Bookmarks

Ðiều Chỉnh
Xếp Bài

Quuyền Hạn Của Bạn
You may not post new threads
You may not post replies
You may not post attachments
You may not edit your posts

BB code is Mở
Smilies đang Mở
[IMG] đang Mở
HTML đang Tắt

Chuyển đến


Múi giờ GMT. Hiện tại là 06:23 PM.


Powered by: vBulletin Copyright ©2000-2024, Jelsoft Enterprises Ltd.
Inactive Reminders By mathscope.org
[page compression: 109.25 k/125.41 k (12.89%)]